1answer.
Ask question
Login Signup
Ask question
All categories
  • English
  • Mathematics
  • Social Studies
  • Business
  • History
  • Health
  • Geography
  • Biology
  • Physics
  • Chemistry
  • Computers and Technology
  • Arts
  • World Languages
  • Spanish
  • French
  • German
  • Advanced Placement (AP)
  • SAT
  • Medicine
  • Law
  • Engineering
sergeinik [125]
2 years ago
7

What is the net force acting on Hector and the toboggan?

Physics
1 answer:
prohojiy [21]2 years ago
7 0

Answer:

C.490N

Explanation:

You might be interested in
Any person for the job he applied Force moment or torque​
VLD [36.1K]

Answer:

i think torque is the answer not confirmed

7 0
2 years ago
what occurs in a chemical reaction? A. Reactants are formed without chemical bonds being broken B. Reactants are formed from pro
Ann [662]
The answer is D. Products are formed from reactants by the breaking and forming of new bonds. 
3 0
2 years ago
Read 2 more answers
In an experiment what are all the parts of an experiment that remain unchanged are called<br>​
stira [4]
The answer for this question is Control Variable because it doesn’t change throughout the experiment.
3 0
3 years ago
The Robinson projection map is considered very useful because
Vladimir79 [104]
Hello,
   The question states: <span>The Robinson projection map is considered very useful because...
The answer is \boxed{because \ most \ distances, \ sizes, \ and \ shapes \ are \ accurate.}

Hope this helped!

~FoodJunky
</span>
7 0
3 years ago
A rod extending between x = 0 and x = 13.0 cm has uniform cross-sectional area A = 8.00 cm2. Its density increases steadily betw
mezya [45]

Answer:

The mass is  m  = 3.45408 kg

Explanation:

From the question we are told that

    The extension  of the rod is from , \   x_1 = 0 \to x_2 = 13.0

     The area is  A =  8.0 cm^2

      The density increase as follows from  \  \rho_1 =2.5 g/cm^2 \to  \rho_2= 19.0 g/cm^3

    The equation  \rho =  B + Cx

at  x_1= 0  \rho_1 =2.5 g/cm^2

So

      2.5  =  B  + 0

=>  B =2.5

    So at x_2 = 13.0 ,  \rho_2= 19.0 g/cm^3

So

            19.0 = 2.5 + C(13)

       =>   C = 1.27

Now  

       m  =  8   \int\limits^{13}_{0} {2.5 + 1.27x} \, dx

      m  =  8   [{2.5 +\frac{ 1.27x^2}{2} } ]\left  | 13} \atop {0}} \right.

      m  =  8   [{2.5 +\frac{ 1.27(13)^2}{2} } ]

      m  = 3454.08 g

        m  = 3.45408 kg

         

8 0
2 years ago
Other questions:
  • An object is 25.0 cm from the mirror, its height is 5.0 cm, and focal length is 8.0 cm. What is the distance from the image to t
    15·2 answers
  • Which of the following tends to increase in a system?
    9·1 answer
  • Op- amp as integrater ​
    9·1 answer
  • A "homemade" capacitor can be built by placing a sheet of paper between two foil. Imagine that the sheet of paper is 21 cm x 30
    8·1 answer
  • For an object producing a thermal spectrum, a higher temperature causes the spectrum to have ___________. a. a peak intensity lo
    13·1 answer
  • Why is it impossible to create a perpetual motion machine?
    14·1 answer
  • When does a falling object reach terminal velocity?
    8·1 answer
  • In a certain ideal heat engine, 10.00 kJ of heat is withdrawn from the hot source at 273 K and 3.00 kJ of work is generated. Wha
    6·2 answers
  • bakit itinuring angkop na angkop ang mga nilalamab ng ibong adarna sa kalinangan at kulturang pilipino​
    8·1 answer
  • 3.- Un barco que tiene el motor averiado es ayudado por dos remolcadores para entrar en el puerto. Cada uno de ellos tira de él
    11·1 answer
Add answer
Login
Not registered? Fast signup
Signup
Login Signup
Ask question!